Proving a function is continuous

Click For Summary
To prove that a function is continuous, it is necessary to show that the inverse image of any open set in the codomain is also open in the domain. The discussion focuses on demonstrating that if B is an upper ideal in the topology Ty on Y, then the inverse image f^{-1}(B) must belong to the topology Tx on X. The participant is attempting to establish this by considering elements of B and their relationships within the ordered set. They express uncertainty about how to connect the definitions of upper ideals and finite topological spaces to reach the conclusion. Clarification on the steps to prove continuity is sought, emphasizing the need to show that if x is in the inverse image of B, then any y greater than or equal to x must also be in that inverse image.
Raziel2701
Messages
128
Reaction score
0

Homework Statement


Here it is in all its glory: http://imgur.com/w08cp

Homework Equations


Here is a definition on what an upper ideal is: http://imgur.com/ZILjW
Here's what a finite topological space is: http://imgur.com/tBGTn

The Attempt at a Solution


From what I gather, I want to show that if B\in Ty, then the inverse image of B is an element of Tx.

From there I started by letting B\in Ty. Ty is a topology on Y and it is the set of upper ideals in (Y,\leq). Thus for b\in B\in Ty, and y \in Y, b \leq y so y \in Ty.

But I'm not seeing it, I don't know how one definition gets me to another step. Was I even in the right track?
 
Physics news on Phys.org
Well, let's look at what you need to prove. Take B\in \mathcal{T}_Y.
You need to prove that f^{-1}(B)\in \mathcal{T}_X, thus you need to show that, given x\in f^{-1}(B) and x\leq y, that y\in f^{-1}(B)...
 
Question: A clock's minute hand has length 4 and its hour hand has length 3. What is the distance between the tips at the moment when it is increasing most rapidly?(Putnam Exam Question) Answer: Making assumption that both the hands moves at constant angular velocities, the answer is ## \sqrt{7} .## But don't you think this assumption is somewhat doubtful and wrong?

Similar threads

Replies
10
Views
3K
  • · Replies 12 ·
Replies
12
Views
2K
  • · Replies 40 ·
2
Replies
40
Views
4K
  • · Replies 1 ·
Replies
1
Views
1K
  • · Replies 20 ·
Replies
20
Views
4K
  • · Replies 2 ·
Replies
2
Views
2K
  • · Replies 20 ·
Replies
20
Views
3K
  • · Replies 5 ·
Replies
5
Views
2K
Replies
5
Views
2K
  • · Replies 5 ·
Replies
5
Views
1K